Epsilon-limit proof for real number sequences

In summary: Thanks! :)I am sorry to have to go, but I must. I am not sure why you are stuck. I am not sure about the 2. Is it 2 times something or a number squared? I tried to explain how to find N1 and left you with a little bit to do. I am sorry I cannot help you any further.
  • #1
kingwinner
1,270
0

Homework Statement


Let {an} be a sequence of real numbers.
Suppose an->L as n->∞. Prove that [(a1+a2+...+an)/n] ->L as n->∞.

Homework Equations


N/A

The Attempt at a Solution


By definition:
an->L iff
for all ε>0, there exists an integer N such that n≥N => |an - L|< ε.

Given ε>0.
I start with |[(a1+a2+...+an)/n] - L|. But I cannot think of a way to link this with |an - L| so I have no idea how to continue.

Any help is appreciated! :)
 
Physics news on Phys.org
  • #2
I think we may have to use the triangle inequality, but still I have no idea how to get into the exact the same form as |an - L|.

Could someone help me, please?
 
Last edited:
  • #3
LCKurtz said:
Given [itex]\varepsilon > 0[/itex] there is N such that if n > N, then you can make

[tex]|\frac{a_1 + ... + a_n}{n}| < \frac \varepsilon 4[/tex]

I don't get this part.

In the hypothesis, we suppose an->L, so:
for all εa>0, there exists an integer N such that n≥N => |an - L|< εa.

So for example:
Given ε>0. We can find N s.t. n≥N => |an-L|< ε/4

But how come you have |(a1+...+an) / n|?

Thanks for your help! :)
 
  • #4
kingwinner said:
I don't get this part.

In the hypothesis, we suppose an->L, so:
for all εa>0, there exists an integer N such that n≥N => |an - L|< εa.

So for example:
Given ε>0. We can find N s.t. n≥N => |an-L|< ε/4

But how come you have |(a1+...+an) / n|?

Thanks for your help! :)

It isn't. I must have been in a fog when I posted that and I have deleted it. Be back a little later.
 
  • #5
Given [itex]\varepsilon > 0[/itex] pick N such that if n > N then [itex]|a_n - L| < \varepsilon/ 4[/itex]. Now if n > N

[tex]\left|\frac{a_1+...+a_n}{n}-L\right| = \left|\frac{a_1+...+a_n}{n}-\frac {nL}{n}\right|= \left|\frac{(a_1-L)+...+(a_n-L)} n\right|[/tex]

[tex]= \left| \frac{(a_1-L)+...+(a_N-L)}{n}+\frac{(a_{N+1}-L)+...+(a_n-L)}{n}\right|[/tex]

Now use the triangle inequality. Since N is fixed you can make the first term small in n is large enough. Then work on the second term to make it small. Sorry for the false start earlier.
 
  • #6
LCKurtz said:
Given [itex]\varepsilon > 0[/itex] pick N such that if n > N then [itex]|a_n - L| < \varepsilon/ 4[/itex]. Now if n > N

[tex]\left|\frac{a_1+...+a_n}{n}-L\right| = \left|\frac{a_1+...+a_n}{n}-\frac {nL}{n}\right|= \left|\frac{(a_1-L)+...+(a_n-L)} n\right|[/tex]

[tex]= \left| \frac{(a_1-L)+...+(a_N-L)}{n}+\frac{(a_{N+1}-L)+...+(a_n-L)}{n}\right|[/tex]

Now use the triangle inequality. Since N is fixed you can make the first term small in n is large enough. Then work on the second term to make it small. Sorry for the false start earlier.
1) hmm...why did you choose ε/4 in [itex]|a_n - L| < \varepsilon/ 4[/itex]??

2) At the end, we have to show that:
for all ε>0, there exists an integer M such that
n≥M => |[(a1+a2+...+an)/n] - L|< ε.
But how can we find such an M?

Thanks for your help! :)
 
Last edited:
  • #7
kingwinner said:
hmm...why did you choose ε/4 in [itex]|a_n - L| < \varepsilon/ 4[/itex]??
Because I anticipate adding a couple of small terms together and having the sum come out less than [itex]\varepsilon[/itex].

At the end, we have to show that:
for all ε>0, there exists an integer M such that
n≥M => |[(a1+a2+...+an)/n] - L|< ε.
But how can we find such an M?

Thanks for your help! :)

Have you done the triangle inequality on this yet, so you have two terms to make small? If you can say there exists an N1 such that n > N1 makes the first term small and an N2 such that n > N2 makes the second term small, then if you take Nmax = max{N1,N2,N}, wouldn't that do it? Of course, you have to fill in the arguments why you can pick these N's.
 
  • #8
LCKurtz said:
Because I anticipate adding a couple of small terms together and having the sum come out less than [itex]\varepsilon[/itex].



Have you done the triangle inequality on this yet, so you have two terms to make small? If you can say there exists an N1 such that n > N1 makes the first term small and an N2 such that n > N2 makes the second term small, then if you take Nmax = max{N1,N2,N}, wouldn't that do it? Of course, you have to fill in the arguments why you can pick these N's.

So after using the triangle inequality (which will break it into 2 terms), for the first term I have to find N1 such that n≥N1 => |[(a1 -L)+...(aN -L)]/n| < ε/2

To find this N1, I think I have to relate |[(a1 -L)+...(aN -L)]/n| to |an - L|, but how is this possible?? I can't think of a way of doing it...

Thanks!
 
  • #9
N is a fixed, finite, number. There are finitely many terms in the numerator and you have n in the denominator. Just pick n big enough...
 
  • #10
LCKurtz said:
N is a fixed, finite, number. There are finitely many terms in the numerator and you have n in the denominator. Just pick n big enough...
I get the intuitive idea, but just like every epsilon-limit proof of this type, we have to actually find a specific N1 which works, right? And I'm stuck on this step...
 
  • #11
kingwinner said:
I get the intuitive idea, but just like every epsilon-limit proof of this type, we have to actually find a specific N1 which works, right? And I'm stuck on this step...

So solve your inequality |[(a1 -L)+...(aN -L)]/n| < ε/2 for n to see how large it must be.
 
  • #12
LCKurtz said:
So solve your inequality |[(a1 -L)+...(aN -L)]/n| < ε/2 for n to see how large it must be.

OK. I think it is n> 2 |(a1-L)+...+(aN-L)| / ε.
So I think we can take N1 = 2 |(a1-L)+...+(aN-L)| / ε, is this correct?
But on top of N depending on ε, N also depends on the terms in the sequence, is this OK? (i.e. is this allowed?)
 
  • #13
kingwinner said:
OK. I think it is n> 2 |(a1-L)+...+(aN-L)| / ε.
So I think we can take N1 = 2 |(a1-L)+...+(aN-L)| / ε, is this correct?
But on top of N depending on ε, N also depends on the terms in the sequence, is this OK? (i.e. is this allowed?)

Everything depends on the terms of the sequence. You wouldn't expect the same N to work for all sequences, would you?

Remember you might have to take N1 slightly larger than that because that might not be an integer.
 
  • #14
So I guess I will take N1 to be the smallest integer greater than or equal to 2 |(a1-L)+...+(aN-L)| / ε.


Any hints for the second term? (sorry I'm asking it but I really have no clue...)

|[aN+1 - L +...+ (an-1-L) +(an-L)] / n|

Here there are terms like an, an-1, etc. The "n" pops up everyhwere, so I don't think I can possibly solve for n in the inequality just like I did before.
 
  • #15
The sequence an converges therefore it is bounded. So every an<M where M is some number. So the sum becomes less that (M-L) ...n-N+1 times over n and from there go on.
 
  • #16
r.a.c. said:
The sequence an converges therefore it is bounded. So every an<M where M is some number. So the sum becomes less that (M-L) ...n-N+1 times over n and from there go on.
|[aN+1 - L +...+ (an-1-L) +(an-L)] / n|
≤|(n-N)(M-L)/n|

I think it should be n-N instead of n-N+1?
n-(N+1)+1 = n-N
 
  • #17
kingwinner said:
So I guess I will take N1 to be the smallest integer greater than or equal to 2 |(a1-L)+...+(aN-L)| / ε.


Any hints for the second term? (sorry I'm asking it but I really have no clue...)

|[aN+1 - L +...+ (an-1-L) +(an-L)] / n|

Here there are terms like an, an-1, etc. The "n" pops up everyhwere, so I don't think I can possibly solve for n in the inequality just like I did before.

Somewhere in this proof you need to use the fact that the original sequence converges to L. Remember at the beginning where you had that if n > N,

[itex]
|a_n - L| < \varepsilon/ 4
[/itex]

Use that now.
 
  • #18
LCKurtz said:
Somewhere in this proof you need to use the fact that the original sequence converges to L. Remember at the beginning where you had that if n > N,

[itex]
|a_n - L| < \varepsilon/ 4
[/itex]

Use that now.

Do I have to use the fact that any convergence sequence is bounded?

But how should I handle the terms like (an-1-L) in the numerator?

thanks.
 
  • #19
kingwinner said:
Do I have to use the fact that any convergence sequence is bounded?

But how should I handle the terms like (an-1-L) in the numerator?

thanks.

I fear you are getting lost in the subscripts. In the expression:

|[aN+1 - L +...+ (an-1-L) +(an-L)] / n|

all the subscripts are greater than N. So every one of them satisfies the inequality:

[itex]

|a_k - L| < \varepsilon/ 4

[/itex]

for whatever value from N+1 to n that k takes. Use that.
 
  • #20
LCKurtz said:
I fear you are getting lost in the subscripts. In the expression:

|[aN+1 - L +...+ (an-1-L) +(an-L)] / n|

all the subscripts are greater than N. So every one of them satisfies the inequality:

[itex]

|a_k - L| < \varepsilon/ 4

[/itex]

for whatever value from N+1 to n that k takes. Use that.

So should I break |[(aN+1 - L) +...+ (an-1-L) +(an-L)] / n| using the triangle inequality into n-N terms?
|[(aN+1 - L) +...+ (an-1-L) +(an-L)] / n| <= (1/n)[|aN+1 - L| + ...+ |an-1-L| + |an-L|] ?

But I don't think it will work...
 
  • #21
kingwinner said:
So should I break |[(aN+1 - L) +...+ (an-1-L) +(an-L)] / n| using the triangle inequality into n-N terms?
|[(aN+1 - L) +...+ (an-1-L) +(an-L)] / n| <= (1/n)[|aN+1 - L| + ...+ |an-1-L| + |an-L|] ?

But I don't think it will work...

You won't know until you try. How big are the terms? How many terms are there?
 
  • #22
LCKurtz said:
You won't know until you try. How big are the terms? How many terms are there?

There are n-N terms.

|[(aN+1 - L) +...+ (an-1-L) +(an-L)] / n| <= (1/n)[|aN+1 - L| + ...+ |an-1-L| + |an-L|] < (1/n)(n-N)ε/4

I can't solve for n here as I did for the first part, so I am not sure what to do next...
 
  • #23
How big can

[tex]\left(\frac {n-N}{n}\right)\left(\frac\varepsilon 4\right)[/tex]

be?
 
  • #24
LCKurtz said:
How big can

[tex]\left(\frac {n-N}{n}\right)\left(\frac\varepsilon 4\right)[/tex]

be?
It goes to 0 as n->infinity.

But how can we find a specific N2 which works (just like before when we found N1) in the epsilon-limit proof?
 
  • #25
You aren't looking for another N and we aren't talking about n -> infinity. All you are trying to do is show this sum is small. You have that it is smaller than

[tex]
\left(\frac {n-N}{n}\right)\left(\frac\varepsilon 4\right)
[/tex]

I will ask you again. Is that small? Look at it and think about it.
 
  • #26
LCKurtz said:
You aren't looking for another N and we aren't talking about n -> infinity. All you are trying to do is show this sum is small. You have that it is smaller than

[tex]
\left(\frac {n-N}{n}\right)\left(\frac\varepsilon 4\right)
[/tex]

I will ask you again. Is that small? Look at it and think about it.
I think we can say it's less than ε/4?
 
  • #27
You think?? Are you sure? If the answer is yes, then you should be able to look back at what has been done and see how to determine an N such the if n > N then

[tex]
\left|\frac{a_1+...+a_n}{n}-L\right| < \varepsilon
[/tex]

At least I hope so. Sack time here.

Edit: This morning I noticed a typo, meant [itex]\varepsilon[/itex] above, not [itex]\varepsilon/4[/itex]
 
Last edited:
  • #28
Yes, I pretty sure it's less than or equal to epsilon/4, because N is some positive integer.
So we've have |(a_1 +...+a_n)/n|< ε/2 + ε/4 = 3ε/4 < ε

If you can say there exists an N1 such that n > N1 makes the first term small and an N2 such that n > N2 makes the second term small, then if you take Nmax = max{N1,N2,N}, wouldn't that do it?

But I am a bit confused now. According to what you said above (in the quote), we should be looking for a workable N2. How can we find it?

thanks!
 
  • #29
You just need to review what has been done, keep the various N's straight, and understand it. Given that an converges to L, you are trying to show that

[tex]\frac {a_1 + ... + a_n} n \rightarrow L[/tex].

So this means that given [itex]\varepsilon > 0[/itex], you must find an integer N such that if n > N then

[tex]
\left|\frac{a_1+...+a_n}{n}-L\right| < \varepsilon
[/tex]

As in post #5, you know you can pick N1 such that if n > N1 then [itex] |a_n - L| < \varepsilon/ 4[/itex]

[tex]
\left|\frac{a_1+...+a_n}{n}-L\right| \leq \left| \frac{(a_1-L)+...+(a_{N_1}-L)}{n}+\frac{(a_{N_1+1}-L)+...+(a_n-L)}{n}\right|
[/tex]

And you applied the triangle inequality to that to get:

[tex]
\left|\frac{a_1+...+a_n}{n}-L\right| \leq \left| \frac{(a_1-L)+...+(a_{N_1}-L)}{n}\right|+\left|\frac{(a_{N_1+1}-L)+...+(a_n-L)}{n}\right|
[/tex]

Now, in post #14 you saw now big you had to have n to be to make the first term small. Now let's call that N by the name N2, since we are already using N1 and want to save N for the final one. And remember that if N2 isn't already greater than N1, take it large enough so that it is, so now if n > N2 you have the first term less than [itex]\varepsilon/2[/itex] and [itex] |a_n - L| < \varepsilon/ 4[/itex].

Now use what you know about the second term and you should be able to see the finish line.
 
  • #30
LCKurtz said:
You just need to review what has been done, keep the various N's straight, and understand it. Given that an converges to L, you are trying to show that

[tex]\frac {a_1 + ... + a_n} n \rightarrow L[/tex].

So this means that given [itex]\varepsilon > 0[/itex], you must find an integer N such that if n > N then

[tex]
\left|\frac{a_1+...+a_n}{n}-L\right| < \varepsilon
[/tex]

As in post #5, you know you can pick N1 such that if n > N1 then [itex] |a_n - L| < \varepsilon/ 4[/itex]

[tex]
\left|\frac{a_1+...+a_n}{n}-L\right| \leq \left| \frac{(a_1-L)+...+(a_{N_1}-L)}{n}+\frac{(a_{N_1+1}-L)+...+(a_n-L)}{n}\right|
[/tex]

And you applied the triangle inequality to that to get:

[tex]
\left|\frac{a_1+...+a_n}{n}-L\right| \leq \left| \frac{(a_1-L)+...+(a_{N_1}-L)}{n}\right|+\left|\frac{(a_{N_1+1}-L)+...+(a_n-L)}{n}\right|
[/tex]

Now, in post #14 you saw now big you had to have n to be to make the first term small. Now let's call that N by the name N2, since we are already using N1 and want to save N for the final one. And remember that if N2 isn't already greater than N1, take it large enough so that it is, so now if n > N2 you have the first term less than [itex]\varepsilon/2[/itex] and [itex] |a_n - L| < \varepsilon/ 4[/itex].

Now use what you know about the second term and you should be able to see the finish line.
I think this would work.
Take N=max{N1,N2}
where N1 is as you defined
and N2 = the smallest integer greater than or equal to 2 |(a1-L)+...+(aN-L)| / ε.
If n > N, then |[(a1+a2+...+an)/n] - L|< ε.


However, according to what you said in post #7, we should take N to be the max of THREE things, i.e. N=max{N1,N2,N3}. Why? What is N3? I don't understand where this third restriction comes from.
I believe |[(aN+1 - L) +...+ (an-1-L) +(an-L)] / n| <= (1/n)[|aN+1 - L| + ...+ |an-1-L| + |an-L|] < (1/n)(n-N)ε/4 < ε/4 is always true, isn't it? So what is N3?

Thanks for your time and patience! (this topic is really new to me)
 
  • #31
kingwinner said:
I think this would work.
Take N=max{N1,N2}
where N1 is as you defined
and N2 = the smallest integer greater than or equal to 2 |(a1-L)+...+(aN-L)| / ε.
If n > N, then |[(a1+a2+...+an)/n] - L|< ε.

That is correct.

However, according to what you said in post #7, we should take N to be the max of THREE things, i.e. N=max{N1,N2,N3}. Why? What is N3? I don't understand where this third restriction comes from.
I believe |[(aN+1 - L) +...+ (an-1-L) +(an-L)] / n| <= (1/n)[|aN+1 - L| + ...+ |an-1-L| + |an-L|] < (1/n)(n-N)ε/4 < ε/4 is always true, isn't it? So what is N3?
In our back-and-forth we may have called N different things; that's why I summarized it in post #29. We don't need an N3.
 

1. What is an epsilon-limit proof for real number sequences?

An epsilon-limit proof for real number sequences is a mathematical technique used to formally prove that a given sequence of real numbers converges to a specific limit, given an arbitrary small value of epsilon. It involves using the definition of convergence and manipulating it to show that the sequence eventually gets arbitrarily close to the limit.

2. Why is an epsilon-limit proof important?

An epsilon-limit proof is important because it provides a rigorous and systematic way to prove that a sequence of real numbers converges to a specific limit. This is a fundamental concept in real analysis and is used in many other areas of mathematics and science.

3. What is the role of epsilon in an epsilon-limit proof?

Epsilon is a small positive number that is used in the definition of convergence. In an epsilon-limit proof, it represents the desired level of precision or closeness to the limit that the sequence must eventually reach. The smaller the value of epsilon, the closer the sequence must get to the limit, making the proof more rigorous.

4. Can an epsilon-limit proof be used for all real number sequences?

Yes, an epsilon-limit proof can be used for all real number sequences, as long as the sequence is convergent. This means that the sequence approaches a specific limit as the number of terms in the sequence increases. If a sequence is not convergent, an epsilon-limit proof cannot be used.

5. What are some common techniques used in an epsilon-limit proof?

Some common techniques used in an epsilon-limit proof include the definition of convergence, the triangle inequality, and the use of the Archimedean property. Other techniques may also be used depending on the specific sequence and limit being considered.

Similar threads

  • Calculus and Beyond Homework Help
Replies
8
Views
2K
  • Calculus and Beyond Homework Help
Replies
13
Views
2K
  • Calculus and Beyond Homework Help
Replies
5
Views
1K
  • Calculus and Beyond Homework Help
Replies
13
Views
2K
  • Calculus and Beyond Homework Help
Replies
34
Views
2K
  • Calculus and Beyond Homework Help
Replies
1
Views
715
  • Calculus and Beyond Homework Help
Replies
2
Views
842
  • Calculus and Beyond Homework Help
Replies
3
Views
843
  • Calculus and Beyond Homework Help
Replies
14
Views
4K
Replies
1
Views
571
Back
Top